Difference between revisions of "2016 AMC 10A Problems/Problem 5"
(→Video Solution) |
Technodoggo (talk | contribs) |
||
(2 intermediate revisions by 2 users not shown) | |||
Line 13: | Line 13: | ||
As seen in the first solution, we end up with <math>12x^3</math>. Taking the answer choices and dividing by <math>12</math>, we get <math>(A) 4</math>, | As seen in the first solution, we end up with <math>12x^3</math>. Taking the answer choices and dividing by <math>12</math>, we get <math>(A) 4</math>, | ||
<math>(B) 4 \frac{2}{3}</math>, <math>(C) 5 \frac{1}{3}</math>, <math>(D) 8</math>, <math>(E) 12</math> and the final answer has to equal <math>x^3</math>. The only answer choice that works is <math>(D)</math>. | <math>(B) 4 \frac{2}{3}</math>, <math>(C) 5 \frac{1}{3}</math>, <math>(D) 8</math>, <math>(E) 12</math> and the final answer has to equal <math>x^3</math>. The only answer choice that works is <math>(D)</math>. | ||
+ | |||
+ | ==Solution 3 (quick)== | ||
+ | |||
+ | Clearly, the volume is a multiple of <math>3\cdot4=12</math>. Since <math>12</math> itself is not an option, the next possibility is <math>2\cdot6\cdot8=\boxed{\textbf{(D)}~96}</math>. | ||
+ | |||
+ | ~Technodoggo | ||
==Video Solution (HOW TO THINK CREATIVELY!!!)== | ==Video Solution (HOW TO THINK CREATIVELY!!!)== | ||
Line 28: | Line 34: | ||
~savannahsolver | ~savannahsolver | ||
+ | |||
+ | https://www.youtube.com/watch?v=4zTfNAWEzys | ||
+ | |||
+ | ~IBHishere | ||
==See Also== | ==See Also== | ||
{{AMC10 box|year=2016|ab=A|num-b=4|num-a=6}} | {{AMC10 box|year=2016|ab=A|num-b=4|num-a=6}} | ||
{{MAA Notice}} | {{MAA Notice}} |
Latest revision as of 23:23, 5 September 2024
Contents
Problem
A rectangular box has integer side lengths in the ratio . Which of the following could be the volume of the box?
Solution 1
Let the smallest side length be . Then the volume is . If , then
Solution 2
As seen in the first solution, we end up with . Taking the answer choices and dividing by , we get , , , , and the final answer has to equal . The only answer choice that works is .
Solution 3 (quick)
Clearly, the volume is a multiple of . Since itself is not an option, the next possibility is .
~Technodoggo
Video Solution (HOW TO THINK CREATIVELY!!!)
https://youtu.be/bc-somFWrbg
~Education, the Study of Everything
Video Solution
https://youtu.be/VIt6LnkV4_w?t=512
~savannahsolver
https://www.youtube.com/watch?v=4zTfNAWEzys
~IBHishere
See Also
2016 AMC 10A (Problems • Answer Key • Resources) | ||
Preceded by Problem 4 |
Followed by Problem 6 | |
1 • 2 • 3 • 4 • 5 • 6 • 7 • 8 • 9 • 10 • 11 • 12 • 13 • 14 • 15 • 16 • 17 • 18 • 19 • 20 • 21 • 22 • 23 • 24 • 25 | ||
All AMC 10 Problems and Solutions |
The problems on this page are copyrighted by the Mathematical Association of America's American Mathematics Competitions.